Difference between revisions of "2022 AMC 12B Problems/Problem 5"

(Created page with "== Problem == The point <math>(-1, -2)</math> is rotated <math>270^{\circ}</math> counterclockwise about the point <math>(3, 1)</math>. What are the coordinates of its new pos...")
(No difference)

Revision as of 18:19, 17 November 2022

Problem

The point $(-1, -2)$ is rotated $270^{\circ}$ counterclockwise about the point $(3, 1)$. What are the coordinates of its new position?

$\textbf{(A)}\ (-3, -4) \qquad \textbf{(B)}\ (0,5) \qquad \textbf{(C)}\ (2,-1) \qquad \textbf{(D)}\ (4,3) \qquad \textbf{(E)}\ (6,-3)$